Can we attack the premise in weakening argument type ques.?

This topic has expert replies
Junior | Next Rank: 30 Posts
Posts: 21
Joined: Sun Aug 28, 2011 9:13 am
Thanked: 1 times
The gray squirrel, introduced into local woodlands ten years ago, threatens the indigenous population of an endangered owl species, because the squirrels habitual stripping of tree bark destroys the trees in which the owls nest. Some local officials have advocated setting out poison for the gray squirrels. The officials argue that this measure, while eliminating the squirrels, would pose no threat to the owl population, since the poison would be placed in containers accessible only to squirrels and other rodents.

Which one of the following, if true, most calls into question the officials argument?

(A) One of the species whose members are likely to eat the poison is the red squirrel, a species on which owls do not prey.
(B) The owls whose nesting sites are currently being destroyed by the gray squirrels feed primarily on rodents.
(C) No indigenous population of any other bind species apart from the endangered owls is threatened by the gray squirrels.
(D) The owls that tare threatened build their nests in the tops of trees, but the gray squirrels strip away back from the trunks.
(E) The officials plan entails adding the poison to food sources that are usually eaten by rodents but not by other animals.

OA: B

I could get the OA but my concern is about D, the second best option. Had B not been an option would D qualify to weaken the argument. As D directly attacks the evidence.

Also it is ok for a answer choice to directly attack a premise in GMAT ?

User avatar
Legendary Member
Posts: 582
Joined: Tue Mar 08, 2011 12:48 am
Thanked: 61 times
Followed by:6 members
GMAT Score:740

by force5 » Fri Sep 16, 2011 12:01 pm
gmattaker20...... i didn't understand your question? ... why do you even want to think about a back up answer if it were not B :-)

when you have the best answer don't think about the second best answer or do research on the other options. BTW how on earth is D weakening anything. I think we is not even relevant to the argument.

remember question never asks you to choose the best and second best option.

Master | Next Rank: 500 Posts
Posts: 416
Joined: Thu Jul 28, 2011 12:48 am
Thanked: 28 times
Followed by:6 members

by gunjan1208 » Fri Sep 16, 2011 9:14 pm
Hi Force 5,

Imagine a case where GMAT hides B and give some other coice which is very related to D. Thus I would recommend the thought proces for sure.

To answer the question --> We are already told that squirrel tear apart the bark of the tree and due to to that one of the owl species is threatened. Then D does not really become the argument for saving owl who live at the top. There may be case that tree can not sustain tempratures and sue to that tree is ruined/gone.

This is why I did not choose D....But I would really be intreted to know from other's point of view as well.

thank you!!

Junior | Next Rank: 30 Posts
Posts: 21
Joined: Sun Aug 28, 2011 9:13 am
Thanked: 1 times

by gmattaker20 » Fri Sep 16, 2011 11:16 pm
Gunjan what I though while considering choice D is : It attacks the premise in the argument which states that "the squirrels habitual stripping of tree bark destroys the trees in which the owls nest". Choice D says the barks stripped away from the trunks would not affect owl as they live in the tops of trees. So it is very well within the scope of argument and evidently weakens it too.

Master | Next Rank: 500 Posts
Posts: 416
Joined: Thu Jul 28, 2011 12:48 am
Thanked: 28 times
Followed by:6 members

by gunjan1208 » Sat Sep 17, 2011 12:42 am
Hi GMATTaker20,

Beautiful..See the following excerpt from the statement:-

The officials argue that this measure, while eliminating the squirrels, would pose no threat to the owl population, since the poison would be placed in containers accessible only to squirrels and other rodents.

It is decided already that Squirrels are threat. What we need to find a reasoning for is why the option suggested is not very good strategy though. And in that case, D does not affect owls. While if B happens, rodents are gone and owls will starve.

Nonetheless, I liked the way you presented it. Thanks for the new angle.

Junior | Next Rank: 30 Posts
Posts: 21
Joined: Sun Aug 28, 2011 9:13 am
Thanked: 1 times

by gmattaker20 » Sat Sep 17, 2011 1:24 am
Thanks for the reply Gunjan and I think I got your point. What you saying is that the correct choice ought to attack the conclusion which is "squirrels elimination pose no threat to owl" by negating it.

Your logic attacks the conclusion (as in option B) by stating "squirrels elimination pose threat to owl" whereas option D attacks the argument by concluding "squirrels elimination pose no threat to owl" but invalidating the premise. See the difference in logical structure. Both attacks the argument but differently.

I think why D is wrong option because it refutes the stated premise which is "the squirrels habitual stripping of tree bark destroys the trees". Though I am still not sure for the elimination of D. Any expert views on this ?

User avatar
GMAT Instructor
Posts: 2193
Joined: Mon Feb 22, 2010 6:30 pm
Location: Vermont and Boston, MA
Thanked: 1186 times
Followed by:512 members
GMAT Score:770

by David@VeritasPrep » Wed Sep 21, 2011 8:19 am
gmattaker20 -

Choice D does not actually attack the premise. Please remember that just as individual numbers matter in the quantitative portion of the exam, individual words are crucial in critical reasoning.

The stimulus states "The gray squirrel, introduced into local woodlands ten years ago, threatens the indigenous population of an endangered owl species, because the squirrels habitual stripping of tree bark destroys the trees in which the owls nest."

Notice the portion that I have placed in bold. The squirrels destroy the trees. It does not matter if they destroy the trees by attacking the lower portion of the trees or the upper part. In the end, the trees are destroyed.

So when D states, "(D) The owls that are threatened build their nests in the tops of trees, but the gray squirrels strip away back from the trunks." This does nothing to contradict the premise that the squirrels destroy the trees. If you are standing at the top of a ladder you would not be okay if I was sawing away at the bottom of the ladder!

To answer your more general question, NO you cannot attack a premise in order to weaken a conclusion. You want to really attack the link between the premise and the conclusion.

Here is a useful little template to use for weaken questions.

Say to yourself,

"Despite the fact that (insert the premise) is true, (insert conclusion) is not necessarily true).

So in this case "Despite the fact that it is true that the poison would only be accessible to rodents, it is not necessarily true that the poison poses no threat to the owls."

So you are looking for the conclusion to be weakened, even though the premises may still be true. It is that link between premise and conclusion that you should focus on.

Happy studying!
Veritas Prep | GMAT Instructor

Veritas Prep Reviews
Save $100 off any live Veritas Prep GMAT Course

User avatar
Master | Next Rank: 500 Posts
Posts: 496
Joined: Tue Jun 07, 2011 5:34 am
Thanked: 38 times
Followed by:1 members

by sl750 » Wed Sep 21, 2011 8:38 am
Choice B
The goal is to kill squirrels and rodents, but if owls feed on these rodents as their primary source of food, the owls will be harmed too

Junior | Next Rank: 30 Posts
Posts: 21
Joined: Sun Aug 28, 2011 9:13 am
Thanked: 1 times

by gmattaker20 » Wed Sep 21, 2011 11:19 pm
Thanks David for the elaborative explanation. Even earlier I too thought on the same lines but got carried away in logical deductions.

And as far as you mentioned that an answer choice can never attack a premise I came across an example which i would like to share. Although, it is very rare in a weakening question to attack a premise, I still think it is rare but not incorrect.

The function of government is to satisfy the genuine wants of the masses, and government cannot satisfy those wants unless it is informed about what those wants are. Freedom of speech ensures that such information will reach the ears of government officials. Therefore, freedom of speech is indispensable for a healthy state.

Example:

Which one of the following, if true, would NOT undermine the conclusion of the argument?

(A) People most often do not know what they genuinely want.
(B) Freedom of speech tends ultimately to undermine social order, and social order is a prerequisite for satisfying the wants of the masses.
(C) The proper function of government is not to satisfy wants, but to provide equality of opportunity.
(D) Freedom of speech is not sufficient for satisfying the wants of the masses: social order is necessary as well.
(E) Rulers already know what the people want.

OA: D

Here answer choice C negates the premise and is a legitimate weakening choice.

David, please share your views on this example and correct me if I am going wrong. Thanks

User avatar
GMAT Instructor
Posts: 2193
Joined: Mon Feb 22, 2010 6:30 pm
Location: Vermont and Boston, MA
Thanked: 1186 times
Followed by:512 members
GMAT Score:770

by David@VeritasPrep » Thu Sep 22, 2011 8:57 am
Can you give me the source...

I would like to know if it is an official question or at least from a top company so that we know what we are dealing with...

David
Veritas Prep | GMAT Instructor

Veritas Prep Reviews
Save $100 off any live Veritas Prep GMAT Course

Junior | Next Rank: 30 Posts
Posts: 21
Joined: Sun Aug 28, 2011 9:13 am
Thanked: 1 times

by gmattaker20 » Fri Sep 23, 2011 2:08 am
Hi David,

I got this one from a doc which contains past LSAT questions so this question apparently belongs to LSAT.

I tried to google the question and found various links referring to LSAT too.

Here is the link: https://www.google.co.in/#sclient=psy-ab ... 40&bih=764

User avatar
GMAT Instructor
Posts: 2193
Joined: Mon Feb 22, 2010 6:30 pm
Location: Vermont and Boston, MA
Thanked: 1186 times
Followed by:512 members
GMAT Score:770

by David@VeritasPrep » Sat Sep 24, 2011 6:26 pm
So I looked this up and as far as I can tell this is not an LSAT question. My sources are very reliable and if this is an LSAT question it has to be from before 1991, but I doubt that as well. It could also have started life as an LSAT question and in a VERY different form.

Anyway, the fact remains, that on the GMAT (and yes the LSAT too) The premises are to be considered factual and are not supposed to be attacked by the answer choices. It is the linkage between the premise and conclusion that is to be attacked.

After all this is critical reasoning with an emphasis on logic and if I set up the problem that says: "Socrates is a man; All men are mortal; therefore Socrates is mortal" and if I then weakened this by saying "just kidding, Socrates is not a man" that is not in the spirit of critical reasoning.

I will keep looking, a scientist never falls in love with any position but is always looking for new evidence so if I can find an official critical reasoning problem that attacks the premises I will be open to it!!
Veritas Prep | GMAT Instructor

Veritas Prep Reviews
Save $100 off any live Veritas Prep GMAT Course

Junior | Next Rank: 30 Posts
Posts: 21
Joined: Sun Aug 28, 2011 9:13 am
Thanked: 1 times

by gmattaker20 » Sun Sep 25, 2011 10:55 pm
Thanks David for the confirming. Even I had this belief that an answer choice can't attack the premise unless I read it in "Powerscore CR bible".

In this book in chapter six, page 113 the author states that attacking the premise is the most obvious (hence easy to spot) way to weaken an argument and therefore, in practice it is very rare to see such kind of questions. So I thought it is rare but not incorrect. But now that you confirm premise should never be undermined I take that as the final word.

User avatar
GMAT Instructor
Posts: 2193
Joined: Mon Feb 22, 2010 6:30 pm
Location: Vermont and Boston, MA
Thanked: 1186 times
Followed by:512 members
GMAT Score:770

by David@VeritasPrep » Mon Sep 26, 2011 3:29 am
What I said was that you are supposed to treat the premises as true on the GMAT and that I have not seen the Official Question where the premise is undermined.

I am always open to learning something new and will continue to look for this tactic to appear on the GMAT. Until the GMAT starts to offer correct answers that attack the premises your best strategy is to always treat the premises as facts that are absolutely true.

Keep in mind that this does not apply to premises that are not those of the argument or the author. As when an argument says, "some people will tell you to skim on reading comprehension, HOWEVER, because skimming can create a mistaken impression of the passage, this is not the best way to address reading comprehension." Obviously the answers are free to attack everything before the "however" since that is not part of the premises that the argument relies on.
Veritas Prep | GMAT Instructor

Veritas Prep Reviews
Save $100 off any live Veritas Prep GMAT Course

Senior | Next Rank: 100 Posts
Posts: 54
Joined: Tue Aug 30, 2011 12:29 pm
Thanked: 4 times

by theforrestgump » Mon Sep 26, 2011 3:01 pm
gmattaker20 wrote:Thanks David for the confirming. Even I had this belief that an answer choice can't attack the premise unless I read it in "Powerscore CR bible".

In this book in chapter six, page 113 the author states that attacking the premise is the most obvious (hence easy to spot) way to weaken an argument and therefore, in practice it is very rare to see such kind of questions. So I thought it is rare but not incorrect. But now that you confirm premise should never be undermined I take that as the final word.
I read this part in the book as well...but what they meant was what exactly David said...if there is an answer choice attacking the premise, then it is just simple contradicting the premise and hence is very easy to spot...so you can almost not expect such a question...